subject
Mathematics, 28.04.2021 07:40 smann211

Let n be an integer. Tracy claims that-n must be less than 0. To convince Tracy that
his statement is only sometimes true do the
following:
(1) Place n on the number line so that the
value of -n is less than 0.
+
0
(2) Place n on the number line so that the
value of --n is greater than 0.
0

ansver
Answers: 2

Other questions on the subject: Mathematics

image
Mathematics, 21.06.2019 18:30, waterborn7152
Me complete this proof! prove that a quadrilateral is a square. me with the steps for this proof.
Answers: 1
image
Mathematics, 21.06.2019 20:30, kordejah348
10 points? me . its important ‼️‼️
Answers: 1
image
Mathematics, 21.06.2019 21:30, kalieghcook
If t17 = 3 (t5) in an arithmetic progression, find t1 in terms of d.
Answers: 1
image
Mathematics, 21.06.2019 23:00, Pingkay7111
Which geometric principle is used to justify the construction below?
Answers: 1
You know the right answer?
Let n be an integer. Tracy claims that-n must be less than 0. To convince Tracy that
his sta...

Questions in other subjects:

Konu
Mathematics, 20.07.2019 07:00
Konu
Social Studies, 20.07.2019 07:00